Math, asked by neelakshi6882, 1 year ago

If cosec theta - sin theta=m and sec theta - cos theta=n, then prove that (m^n)^2/3 + (mn^2)^2/3 = 1


Sasii: Is that m to the power n to the power 2/3 in the question?

Answers

Answered by MaheswariS
3

\textbf{Given:}

m=cosec\theta-sin\theta

n=sec\theta-cos\theta

\textbf{To prove:}

(m^2n)^\frac{2}{3}+(mn^2)^\frac{2}{3}=1

\textbf{Solution:}

m=cosec\theta-sin\theta

m=\dfrac{1}{sin\theta}-sin\theta

m=\dfrac{1-sin^2\theta}{sin\theta}

m=\dfrac{cos^2\theta}{sin\theta}

n=sec\theta-cos\theta

n=\dfrac{1}{cos\theta}-cos\theta

n=\dfrac{1-cos^2\theta}{cos\theta}

n=\dfrac{sin^2\theta}{cos\theta}

m^2n=\dfrac{cos^4\theta}{sin^2\theta}{\times}\dfrac{sin^2\theta}{cos\theta}

\implies\bf\,m^2n=cos^3\theta

mn^2=\dfrac{cos^2\theta}{sin\theta}{\times}\dfrac{sin^4\theta}{cos^2\theta}

\implies\bf\,mn^2=sin^3\theta

\text{Now,}

(m^2n)^\frac{2}{3}+(mn^2)^\frac{2}{3}

=(cos^3\theta)^\frac{2}{3}+(sin^3\theta)^\frac{2}{3}

=cos^2\theta+sin^2\theta

=1

\therefore\boxed{\bf(m^2n)^\frac{2}{3}+(mn^2)^\frac{2}{3}=1}

Find more:

If sinθ+cosθ=x prove that sin6θ+cos6θ=4-3(x²-1)²/4

https://brainly.in/question/15923140

If l tan A + m sec A = n and l’tan A – m’sec A = n’, then show that (nl’ – ln’/ml’ + lm’)^2 = 1 + (nm’ – mn’/ lm’ + ml’) ^2

https://brainly.in/question/683329

Similar questions